L must be the monument that was begun in 602 if which one of the following is true?

Flavio on April 16, 2017

How is E the right answer?

If L is in 602, wouldn't it force G to be in 601? It's the most restrictive choice that I'm seeing.

Replies
Create a free account to read and take part in forum discussions.

Already have an account? log in

Mehran on April 21, 2017

@Flavio (B) is the trap answer here.

The question is NOT what must occur if L is in 602.

The questions is "L must be in 602 if . . . "

You can have the following scenario, which proves that L does not have to be in 602 if G is in 601:

G/S M L F H

You can also watch a detailed video explanation for this question by tapping the â–¶ icon in the top right hand corner of the screen. Select "Question Explanation."

You can view this question directly from this message board by tapping "View" at the the top of this screen.

Hope this helps! Please let us know if you have any other questions.

Yuliya on October 18, 2019

This explanation is not clear enough.

on August 12, 2020

Nice explanation! I fell for this trap. With practice, I will hopefully catch these in future. The conditions implicit to the question.

on March 25, 2021

The audio for the video is not working for me. Please advise if anyone else is having the same issue. I wanted to hear it explained for this question.

Victoria on May 2, 2021

Hi @fari,

Thank you for letting us know. We are sorry to hear that you are experiencing technical issues. Please direct any support-related issues to our support staff by tapping "support" from the left menu or by calling 855.483.7862 ext. 2 Monday to Friday from 9:00 am to 6:00 pm PT.

The instructors monitor the message boards and specialize in helping with specific LSAT questions and private tutoring.

Thanks and let us know if you have any content-related questions!